Radius of Convergence Power Series

In summary: I think he must have read my work wrong. It was a 10 point problem and I got 4 points. The solution was also for 4 points. I am not sure what happened. I think he may have read the work wrong and he gave me the 4 points he though I got wrong. I am not sure what to do. I don't want to upset him and make him go back over it again. He is a really nice person and I don't want to hurt his feelings. I will try to talk myself into just leaving it alone. This is like the third time I have gotten lower than a B on a problem. It isn't like I am bad
  • #1
xspook
19
0

Homework Statement



Determine the radius of convergence and the interval of convergence og the folling power series:

n=0 to infinity
Ʃ=[itex]\frac{(2x-3)^{n}}{ln(2n+3)}[/itex]

Homework Equations



Ratio Test

The Attempt at a Solution


Well I started with the ratio test but I have no clue where to go with it

[itex]\frac{(2x-3)^{n+1}}{ln(2(n+1)+3)}[/itex] * [itex]\frac{ln(2n+3)}{(2x-3)^{n}}[/itex]

I know that I will be left with the 2x-3 and I can pull that out in front of the limit to use when doing the interval of convergence, but I am lost with the natural log. should I distribute the two and make it

[itex]\frac{(2x-3)^{n+1}}{ln(2n+5)}[/itex] * [itex]\frac{ln(2n+3)}{(2x-3)^{n}}[/itex]

2x-3 Lim as n approaches infinity of : ln(2n+3)-ln(2n+5)??

Any help would be appreciated
 
Physics news on Phys.org
  • #2
xspook said:

Homework Statement



Determine the radius of convergence and the interval of convergence og the folling power series:

n=0 to infinity
Ʃ=[itex]\frac{(2x-3)^{n}}{ln(2n+3)}[/itex]

Homework Equations



Ratio Test

The Attempt at a Solution


Well I started with the ratio test but I have no clue where to go with it

[itex]\frac{(2x-3)^{n+1}}{ln(2(n+1)+3)}[/itex] * [itex]\frac{ln(2n+3)}{(2x-3)^{n}}[/itex]

I know that I will be left with the 2x-3 and I can pull that out in front of the limit to use when doing the interval of convergence, but I am lost with the natural log. should I distribute the two and make it

[itex]\frac{(2x-3)^{n+1}}{ln(2n+5)}[/itex] * [itex]\frac{ln(2n+3)}{(2x-3)^{n}}[/itex]

2x-3 Lim as n approaches infinity of : ln(2n+3)-ln(2n+5)??

Any help would be appreciated

The limit of ##\frac{ln(2n+3)}{ln(2n+5)}## as n goes to infinity is just 1.

Can you continue?

EDIT : It might be informative to wrap your limit in an absolute value.
 
Last edited:
  • #3
Thanks,

let me solve and see what I get
 
  • #4
So I can now set up the interval of convergence to be

-1 < 2x-3 < 1
1 < x < 2

so any number between 1 and two will cause the series to converge

now i have to check endpoints seperately

x:1 Ʃ=[itex]\frac{(2x-3)^{n}}{ln(2n+3)}[/itex]
[itex]\frac{(-1)^{n}}{ln(2n+3)}[/itex] which is an alternating series

the limit is 0 and it is also decreasing

X:2 Ʃ=[itex]\frac{(2x-3)^{n}}{ln(2n+3)}[/itex]
[itex]\frac{(1)^{n}}{ln(2n+3)}[/itex]

this limit is also 0

so the interval is [1,2]?

Radius of convergence would be [itex]\frac{1}{2}[/itex]?

EDIT: when x=2 I feel like I have to do something different for solving for the limit? Or is it correct?
 
  • #5
xspook said:
So I can now set up the interval of convergence to be

-1 < 2x-3 < 1
1 < x < 2

so any number between 1 and two will cause the series to converge

now i have to check endpoints seperately

x:1 Ʃ=[itex]\frac{(2x-3)^{n}}{ln(2n+3)}[/itex]
[itex]\frac{(-1)^{n}}{ln(2n+3)}[/itex] which is an alternating series

the limit is 0 and it is also decreasing

X:2 Ʃ=[itex]\frac{(2x-3)^{n}}{ln(2n+3)}[/itex]
[itex]\frac{(1)^{n}}{ln(2n+3)}[/itex]

this limit is also 0

so the interval is [1,2]?

Radius of convergence would be [itex]\frac{1}{2}[/itex]?

EDIT: when x=2 I feel like I have to do something different for solving for the limit? Or is it correct?

Your radius of convergence is what you get from the calculation of ##lim_{n→∞} |\frac{a_{n+1}}{a_n}| = L##.

If L is finite, then your radius of convergence is 1/L.
If L is 0, then your radius of convergence is ∞.
If L = ∞, then your radius of convergence is 0.

Since you got 1 as the limit, the radius is R = 1/1 = 1. So your series will converge for |2x-3| < 1, aka 1 < x < 2. So you're doing good.

So now to get a concrete interval of convergence, you need to check the endpoints.

At x = 1, it looks like your series is conditionally convergent.

At x = 2, it appears your series diverges. Could you tell me why?

What's your interval now?
 
Last edited:
  • #6
At x = 2, I don't know why it is divergent. I thought it was convergent to zero. If it isn't convergent the interval would become [1,2)
 
  • #7
xspook said:
At x = 2, I don't know why it is divergent. I thought it was convergent to zero. If it isn't convergent the interval would become [1,2)

Okay good about the interval. In the case of x=2, can you compare your series to anything? Think about minimizing the denominator.
 
  • #8
Basically we have

[itex]\frac{1}{ln(2n+3)}[/itex]

The limit of ln(2n+3) is ∞. [itex]\frac{1}{∞}[/itex] = 0

is there something I am missing?
 
  • #9
xspook said:
Basically we have

[itex]\frac{1}{ln(2n+3)}[/itex]

The limit of ln(2n+3) is ∞. [itex]\frac{1}{∞}[/itex] = 0

is there something I am missing?

You're missing the whole picture.

Why are you taking limits? All you have to do is check the convergence of your series at the endpoints.

So when you did x = 1. You saw ##\sum |a_n|## diverged, but ##\sum (-1)^na_n## converges by the alternating series test, so it was conditionally convergent.

Now when x = 2. You don't need the A.S.T. It's quite literally a simple comparison.
 
Last edited:
  • #10
How about:

If I use the comparison test for
[itex]\frac{1}{ln(2n+3)}[/itex] and compare with [itex]\frac{1}{n}[/itex]

[itex]\frac{1}{n}[/itex] is a divergent p series
using the comparison test, [itex]\frac{1}{ln(2n+3)}[/itex] is also divergent?
 
  • #11
xspook said:
How about:

If I use the comparison test for
[itex]\frac{1}{ln(2n+3)}[/itex] and compare with [itex]\frac{1}{n}[/itex]

[itex]\frac{1}{n}[/itex] is a divergent p series
using the comparison test, [itex]\frac{1}{ln(2n+3)}[/itex] is also divergent?

Yes that's the idea.
 
  • #12
Thank you Zondrina!
 
  • #13
My professor gave me a 4 out of 10 possible points...he passed out the solutions to the problems and oddly enough I have the same answer.
 

What is the definition of radius of convergence for a power series?

The radius of convergence for a power series is a value r that represents the distance from the center of the series to the point at which the series converges. In other words, it is the maximum value for x where the series will converge.

How is the radius of convergence calculated?

The radius of convergence is calculated by using the ratio test. This test involves taking the limit of the absolute value of the ratio between consecutive terms in the series. If this limit is less than 1, the series will converge. The radius of convergence is then equal to the reciprocal of this limit.

What is the significance of the radius of convergence?

The radius of convergence is an important concept in power series because it determines the interval of x values for which the series will converge. It also helps to determine the behavior of the series at the endpoints of this interval.

Can the radius of convergence be infinite?

Yes, the radius of convergence can be infinite. This means that the series will converge for all values of x. A series with an infinite radius of convergence is called an entire function.

What happens if the value of x is outside the radius of convergence?

If the value of x is outside the radius of convergence, the series will diverge and will not have a finite sum. This means that the series does not represent a valid function at that point.

Similar threads

  • Calculus and Beyond Homework Help
Replies
1
Views
259
  • Calculus and Beyond Homework Help
Replies
2
Views
189
  • Calculus and Beyond Homework Help
Replies
3
Views
418
  • Calculus and Beyond Homework Help
Replies
5
Views
991
  • Calculus and Beyond Homework Help
Replies
14
Views
1K
  • Calculus and Beyond Homework Help
Replies
4
Views
311
  • Calculus and Beyond Homework Help
Replies
2
Views
711
Replies
8
Views
1K
  • Calculus and Beyond Homework Help
Replies
7
Views
1K
  • Calculus and Beyond Homework Help
Replies
7
Views
709
Back
Top